Đến nội dung

Hình ảnh

Topic: [LTDH] Mỗi ngày hai bất đẳng thức.


  • Chủ đề bị khóa Chủ đề bị khóa
Chủ đề này có 215 trả lời

#41
tritanngo99

tritanngo99

    Đại úy

  • Điều hành viên THPT
  • 1644 Bài viết

Hai bài: Bài 17Bài 18 mọi người đã giải quá chuẩn rồi, nên mình đề xuất hai bài tiếp theo như sau:

Bài 19:Cho $x,y,z>0$. Chứng minh rằng:

$\frac{1}{\sqrt{x^2+y^2+z^2+1}}-\sqrt[3]{\frac{3}{x+y+z}(\frac{x^3}{xy+2yz}+\frac{y^3}{yz+2xz}+\frac{z^3}{xz+2xy})}-\frac{2}{(1+x)(1+y)(1+z)}\le \frac{-3}{4}$.

Bài 20: Cho ba số thực dương $a,b,c$ thỏa mãn: $28(\frac{1}{a^2}+\frac{1}{b^2}+\frac{1}{c^2})=4(\frac{1}{ab}+\frac{1}{bc}+\frac{1}{ca})+2013$.

Tìm GTLN của:

$P=\frac{1}{\sqrt{5a^2+2ab+b^2}}+\frac{1}{\sqrt{5b^2+2bc+c^2}}+\frac{1}{\sqrt{5c^2+2ca+a^2}}$



#42
tpdtthltvp

tpdtthltvp

    Trung úy

  • Điều hành viên THCS
  • 831 Bài viết

Bài 20: Cho ba số thực dương $a,b,c$ thỏa mãn: $28(\frac{1}{a^2}+\frac{1}{b^2}+\frac{1}{c^2})=4(\frac{1}{ab}+\frac{1}{bc}+\frac{1}{ca})+2013$.

Tìm GTLN của:

$P=\frac{1}{\sqrt{5a^2+2ab+b^2}}+\frac{1}{\sqrt{5b^2+2bc+c^2}}+\frac{1}{\sqrt{5c^2+2ca+a^2}}$

Bài 20:

Từ giả thiết suy ra:

$$28(\frac{1}{a}+\frac{1}{b}+\frac{1}{c})^2=60(\frac{1}{ab}+\frac{1}{bc}+\frac{1}{ca})+2013\leq 20(\frac{1}{a}+\frac{1}{b}+\frac{1}{c})^2+2013$$

$$\Leftrightarrow 8(\frac{1}{a}+\frac{1}{b}+\frac{1}{c})^2\leq 2013\Leftrightarrow \frac{1}{a}+\frac{1}{b}+\frac{1}{c}\leq \sqrt{\frac{2013}{8}}$$

Mặt khác ta có:

$$P=\sum \frac{1}{\sqrt{5a^2+2ab+b^2}}=\sum \frac{1}{\sqrt{(2a)^2+(a+b)^2}}\leq \sum \frac{1}{\sqrt{\frac{(3a+b)^2}{2}}}=\sqrt{2}\sum \frac{1}{3a+b}$$

Do đó:

$$P\leq \sqrt{2}\sum \left [ \frac{1}{16}(\frac{3}{a}+\frac{1}{b}) \right ]=\frac{\sqrt{2}}{4}(\frac{1}{a}+\frac{1}{b}+\frac{1}{c})\leq \frac{\sqrt{2}}{4}.\sqrt{\frac{2013}{8}}=\sqrt{\frac{2013}{64}}$$

Vậy $\max P=\sqrt{\frac{2013}{64}}.$ Dấu đẳng thức xảy ra khi $a=b=c=2\sqrt{\frac{6}{671}}$


Bài viết đã được chỉnh sửa nội dung bởi tpdtthltvp: 17-08-2016 - 08:43

$\color{red}{\mathrm{\text{How I wish I could recollect, of circle roud}}}$

$\color{red}{\mathrm{\text{The exact relation Archimede unwound ! }}}$

 


#43
royal1534

royal1534

    Trung úy

  • Điều hành viên THCS
  • 773 Bài viết

Hai bài: Bài 17Bài 18 mọi người đã giải quá chuẩn rồi, nên mình đề xuất hai bài tiếp theo như sau:

Bài 19:Cho $x,y,z>0$. Chứng minh rằng:

$\frac{1}{\sqrt{x^2+y^2+z^2+1}}-\sqrt[3]{\frac{3}{x+y+z}(\frac{x^3}{xy+2yz}+\frac{y^3}{yz+2xz}+\frac{z^3}{xz+2xy})}-\frac{2}{(1+x)(1+y)(1+z)}\le \frac{-3}{4}$.

 

Chém nốt :P.

-------

Áp dụng bất đẳng thức Holder ta có:

 

$\sum \frac{x^3}{xy+2yz} \geq \frac{(x+y+z)^3}{3\sum (xy+2yz)}=\frac{(x+y+z)^3}{9(xy+yz+zx)}$

 

$\Rightarrow \frac{3}{x+y+z}.\sum \frac{x^3}{xy+2yz} \geq \frac{(x+y+z)^2}{3(xy+yz+zx)} \geq 1$

 

Suy ra: $-\sqrt[3]{\frac{3}{x+y+z}.\sum \frac{x^3}{xy+2yz}} \leq -1 $

 

Công việc còn lại ta chỉ cần chứng minh: 

 

$\frac{1}{\sqrt{x^2+y^2+z^2+1}}-\frac{2}{(1+x)(1+y)(1+z)} \leq \frac{1}{4}$

 

Áp dụng bđt Cauchy-Schwarz ta có: 

 

$\frac{1}{\sqrt{x^2+y^2+z^2+1}} \leq \frac{2}{x+y+z+1}$

 

Vậy ta cần chứng minh: $\frac{2}{x+y+z+1}-\frac{2}{(1+x)(1+y)(1+z)} \leq \frac{1}{4}$

 

Đặt $x+1=a, y+1=b, z+1=c$  Vì $x,y,z>0$ suy ra $abc >1$

 

Viết lại bđt cần chứng minh thành:

 

$\frac{2}{a+b+c-2}-\frac{2}{abc} \leq \frac{1}{4}$

 

$\leftrightarrow \frac{1}{a+b+c-2}-\frac{1}{abc} \leq \frac{1}{8}$

 

Quy đồng và biến đổi tương đương. BĐT trên tương đương với:

 

$10abc-8(a+b+c)-abc(a+b+c)+16 \leq 0 $

 

Đổi biến $(abc,a+b+c)=(r,p)$ $(r>1)$

 

Ta cần chứng minh $10r-8p-pr+16 \leq 0 \leftrightarrow 8p+pr-10r-16 \geq 0$

 

Chú ý theo bất đẳng thức AM-GM thì $p \geq 3\sqrt[3]{r}$

 

Tiếp tục đặt $\sqrt[3]{r}=t$ $(t>1)$

 

Thì ta cần chứng minh $24t+3t^4-10t^3-16 \geq 0$

 

$\leftrightarrow (t-2)^2(3t^2+2t-4) \geq 0$: Đúng vì $t > 1$

 

Chứng minh hoàn tất. Dấu '=' xảy ra khi $x=y=z=1$


Bài viết đã được chỉnh sửa nội dung bởi royal1534: 16-08-2016 - 18:22


#44
tritanngo99

tritanngo99

    Đại úy

  • Điều hành viên THPT
  • 1644 Bài viết

Đây là lời giải hai bài 19 và bài 20( cũng gần tương tự lời giải của tpdtthltvp và royal1534):

Lời giải bài 19: Đặt $P=\frac{1}{x+y+z}(\frac{x^3}{y(2z+x)}+\frac{y^3}{z(2x+y)}+\frac{z^3}{x(2y+z)})$.

Ta có: $\frac{x^3}{y(2z+x)}+\frac{y}{3}+\frac{2z+x}{9}\ge x$

$\frac{y^3}{z(2x+y)}+\frac{z}{3}+\frac{2x+y}{9}\ge y$

$\frac{z^3}{x(2y+z)}+\frac{x}{3}+\frac{2y+z}{9}\ge z$.

Cộng vế ta được: $P\ge 1$. Dấu $=$ xảy ra khi $x=y=z=1$.(*)

Đặt $Q=\frac{1}{\sqrt{x^2+y^2+z^2+1}}-\frac{2}{(1+x)(1+y)(1+z)}$

Ta có: $x^2+y^2+z^2+1\ge \frac{1}{2}(x+y)^2+\frac{1}{2}(z+1)^2\ge \frac{1}{4}(x+y+z+1)^2$.

(Vì $a^2+b^2\ge \frac{1}{2}(a+b)^2)$

*$(x+1)(y+1)(z+1)\le (\frac{x+y+z+3}{3})^3$ dấu $=$ xảy ra khi $x=y=z$.

Do đó: $Q\le \frac{2}{x+y+z+1}-\frac{54}{(x+y+z+3)^2}$, đặt $t=x+y+z+1>1$.

Ta được: $Q\le f(t)=\frac{2}{t}-\frac{54}{(t+2)^3}$. Xét hàm số $f(t)$ trên $(1;+\infty)$.

$f'(t)=\frac{-2}{t^2}+\frac{162}{(t+2)^4}=0\iff t=1(l)...or...t=4(n)$. Lập bảng biến thiên ta được: $f(t)\le \frac{1}{4}=f(4)$.

Vậy $Q\le \frac{1}{4}$. Dấu $=$ xảy ra khi $x=y=z=1$(**)

Từ $(*),(**)$ suy ra dpcm.

Lời giải bài 20:

Đặt $(x;y;z)\rightarrow (\frac{1}{a};\frac{1}{b};\frac{1}{c})$.

$\implies 28(x^2+y^2+z^2)=4(xy+yz+zx)+2013\le 4(x^2+y^2+z^2)+2013\implies x^2+y^2+z^2\le \frac{2013}{24}$.

Mặt khác ta lại có: $(x+y+z)^2\le 3(x^2+y^2+z^2)\implies (x+y+z)^2\le \frac{2013}{8}$.

Ta có: $\frac{1}{\sqrt{5a^2+2ab+b^2}}=\frac{1}{\sqrt{4a^2+(a+b)^2}}\le \frac{\sqrt{2}}{3a+b}\le \frac{1}{8\sqrt{2}}(\frac{3}{a}+\frac{1}{b})=\frac{1}{8\sqrt{2}}(3x+y)$.

Tương tự các BDT còn lại cộng lại ta được: $P\le \frac{4}{8\sqrt{2}}(x+y+z)\le \frac{\sqrt{2013}}{8}$.

Đẳng thức xảy ra khi $x=y=z=\frac{\sqrt{4026}}{12}$ hay $a=b=c=\frac{12}{\sqrt{4026}}$ 



#45
tritanngo99

tritanngo99

    Đại úy

  • Điều hành viên THPT
  • 1644 Bài viết

Tiếp theo:

Bài 21: Cho các số thực dương $a,b,c$ thỏa mãn: $a+b+c\ge 12$. Tìm giá trị nhỏ nhất của biểu thức:

$S=\frac{a^3}{\sqrt{ab}+2\sqrt{1+c\sqrt{c}}}+\frac{b^3}{\sqrt{bc}+2\sqrt{1+a\sqrt{a}}}+\frac{c^3}{\sqrt{ca}+2\sqrt{1+b\sqrt{b}}}$.

Bài 22: Cho $a,b,x,y$ là bốn số dương thỏa mãn: $a^5+b^5=2$ và $x,y\le 4$. Hãy tìm GTNN của biểu thức:

$P=\frac{x^2+2y^2+24}{xy(a^2+b^2)}$



#46
VODANH9X

VODANH9X

    Trung sĩ

  • Thành viên
  • 114 Bài viết

Tiếp theo:

Bài 22: Cho $a,b,x,y$ là bốn số dương thỏa mãn: $a^5+b^5=2$ và $x,y\le 4$. Hãy tìm GTNN của biểu thức:

$P=\frac{x^2+2y^2+24}{xy(a^2+b^2)}$

Áp dụng bđt AM-GM:$a^{5}+a^{5}+1+1+1\geq 5a^{2}$

                                  $b^{5}+b^{5}+1+1+1\geq 5b^{2}$

$\Rightarrow 2a^{5}+2b^{5}+6\geq 5(a^{2}+b^{2})\Leftrightarrow a^{2}+b^{2}\leq 2$

$\Rightarrow P\geq \frac{x^{2}+2y^{2}+24}{2xy}=\frac{x}{2y}+\frac{y}{x}+\frac{12}{xy}$

Xét hàm số $f(x)=\frac{x}{2y}+\frac{y}{x}+\frac{12}{xy}$ với $0< x\leq 4$ và y là tham số

$f'(x)=\frac{x^{2}-2y^{2}-24}{2x^2y}\leq \frac{4^{2}-2.0^{2}-24}{2x^{2}y}=\frac{-8}{2x^{2}y}< 0$

Suy ra $f'(x)$ nghịch biến trên $0< x\leq 4$.Do đó $f(x)\geq f(4)$

suy ra $P\geq g(4)=\frac{2}{y}+\frac{y}{4}+\frac{3}{y}=\frac{5}{y}+\frac{y}{4}$

Xét hàm số $g(y)=\frac{5}{y}+\frac{y}{4}$ với $0< y\leq 4$ 

$g'(y)=-\frac{5}{y^{2}}+\frac{1}{4}\leq -\frac{5}{16}+\frac{1}{4}< 0$

Suy ra $g(y)$ nghịch biến trên $0< y\leq 4$.Suy ra $g(y)\geq g(4)=\frac{9}{4}$

Suy ra $P\geq \frac{9}{4}$ dấu bằng xảy ra khi $x=y=4$ và $a=b=1$



#47
phamngochung9a

phamngochung9a

    Sĩ quan

  • Điều hành viên THPT
  • 480 Bài viết

Tiếp theo:

Bài 21: Cho các số thực dương $a,b,c$ thỏa mãn: $a+b+c\ge 12$. Tìm giá trị nhỏ nhất của biểu thức:

$S=\frac{a^3}{\sqrt{ab}+2\sqrt{1+c\sqrt{c}}}+\frac{b^3}{\sqrt{bc}+2\sqrt{1+a\sqrt{a}}}+\frac{c^3}{\sqrt{ca}+2\sqrt{1+b\sqrt{b}}}$.

Theo BĐT $AM-GM$, ta có:

  • $2\sqrt{1+c\sqrt{c}}=2\sqrt{\left ( \sqrt{c}+1 \right )\left ( c-\sqrt{c}+1 \right )}\leq c+2$
  • $\sqrt{ab}\leq \frac{a+b}{2}$

Vậy:

$\sum \frac{a^{3}}{\sqrt{ab}+2\sqrt{1+c\sqrt{c}}}\geq \sum \frac{a^{3}}{\frac{a+b}{2}+c+2}\geq \frac{\left ( a+b+c \right )^{3}}{6\left ( a+b+c+3 \right )}$

 

Đặt $a+b+c=t$, khảo sát hàm số $f\left ( t \right )=\frac{t^{3}}{6\left ( t+3 \right )}$ trên $\left [12;+\infty \right )$ ta được:

$S\geq f\left ( t \right )\geq f\left ( 12 \right )=\frac{96}{5}$

 

Vậy $\min S=\frac{96}{5}\Leftrightarrow a=b=c=4$



#48
tritanngo99

tritanngo99

    Đại úy

  • Điều hành viên THPT
  • 1644 Bài viết

Lời giải bài 21 và bài 22 các bạn VODANH9X và phamngochung9a đã giải chuẩn rồi, mình xin đề xuất hai bài tiếp theo:

Bài 23: Cho các số thực $a,b,c\in [1,2]$. Tìm GTNN của biểu thức:

$P=\frac{(a+b)^2}{c^2+4(ab+bc+ca)}$

Bài 24: Cho các số thực dương $a,b,c$ thỏa mãn: $bc\ge a^2$. Tìm GTNN của biểu thức:

$P=\frac{b^2}{ac}+\frac{c^2}{ab}+\sqrt{\frac{b}{c+a}+\frac{c}{a+b}}$.


Bài viết đã được chỉnh sửa nội dung bởi tritanngo99: 18-08-2016 - 14:23


#49
superpower

superpower

    Sĩ quan

  • Thành viên
  • 492 Bài viết

Lời giải bài 21 và bài 22 các bạn VODANH9X và phamngochung9a đã giải chuẩn rồi, mình xin đề xuất hai bài tiếp theo:

Bài 23: Cho các số thực $a,b,c\in [1,2]$. Tìm GTNN của biểu thức:

$P=\frac{(a+b)^2}{c^2+4(ab+bc+ca)}$

 

Xét hàm số $f(c) = c^2 + 4(ab+bc+ca) $ là hàm đồng biến theo biến $c$

Do đó để $P_{min} $ thì $c $ max

Do đó. ta cần tìm min của biểu thức

$P=\frac{(a+b)^2}{4+4(ab+2a+2b) } $

Mặt khác

Xét hàm số $f(a) = \frac{a^2+2ab+b^2}{(4b+8)a + 4+8b } $

Có $f'(a) = \frac{(4b+8)a^2+ (16b+8)a -4b^3 +8b^2+8b }{[(4b+8)a+4+8b]^2}$

Mà ta có $-4b^3+8b^2+8b > 0 $ (do $b \in [1;2] $)

Do đó $f'(a) >0 $

Tương tự dễ chứng minh $f'(b) >0 $

Mà $a,b \geq 1 => f(a,b) \geq f(1,1) = \frac{1}{6} $

Dấu bằng xảy ra khi $a=b=1; c=2 $ 



#50
phamngochung9a

phamngochung9a

    Sĩ quan

  • Điều hành viên THPT
  • 480 Bài viết

Bài 24: Cho các số thực dương $a,b,c$ thỏa mãn: $bc\le a^2$. Tìm GTNN của biểu thức:

$P=\frac{b^2}{ac}+\frac{c^2}{ab}+\sqrt{\frac{b}{c+a}+\frac{c}{a+b}}$.

Bài 24 hình như có vấn đề rồi anh ơi.

 

Vì $bc\leq a^{2}$ nên ta có thể lấy $a$ lớn tùy ý. Khi cho $a\rightarrow +\infty$ thì 

 

$\frac{b^{2}}{ac}\rightarrow 0;\frac{c^{2}}{ab}\rightarrow 0;\frac{b}{c+a}\rightarrow 0;\frac{c}{a+b}\rightarrow 0\\\Rightarrow P\rightarrow 0$ tức là lấy $a$ càng lớn $b,c$ giữ nguyên thì $P$ càng nhỏ và dần đến $0$.

 

 

Chẳng hạn, lấy $a=10^{9};b=1;c=1$ thì ta có: $P= 0,000044723\approx 0$



#51
MrS

MrS

    Binh nhất

  • Thành viên
  • 29 Bài viết

Và tiếp theo là hai bài sau:

Bài 3: Cho $a,b,c$ là các số thực dương thỏa mãn: $a+b+c=3$. Chứng minh rằng: $3(a^2+b^2+c^2)+4abc\ge 13$.

Bài 4: Cho $x,y,z>1$ thỏa mãn: $x+y+z=xyz$. Tìm min: $X=\frac{x-2}{z^2}+\frac{y-2}{x^2}+\frac{z-2}{y^2}$.

Bài 3: Giải bằng dồn biến: $Giả sử a=min(a,b,c)\Rightarrow a\leq 1$

Đặt $f(a,b,c)=3(a^2+b^2+c^2)+4abc-13$

Đặt t=(b+c)/2 xét $f(a,b,c)- f(a,t,t)= (3/2-a)(b-c)^2\geq 0$ do $a\leq 1$ 

$f(a,t,t)=3(a^2+2t^2)+4at^2-13$, ta có a+2t=3 nên a=3-2t, thế vào và rút gọn ta được: $2(t-1)^2(7-4t)\geq 0$ do $t=\frac{b+c}{2}<\frac{a+b+c}{2}=\frac{3}{2}$ nên 7-4t>0

BĐT được chứng minh. Đẳng thức xảy ra khi a=b=c=1 



#52
tritanngo99

tritanngo99

    Đại úy

  • Điều hành viên THPT
  • 1644 Bài viết

Dưới đây là lời giải bài 23 và bài 24:

Lời giải bài 23:

Ta có: $(b+c)^2\ge 4bc\ge 4a^2\iff \frac{b+c}{a}\ge 2$; $\frac{b^2}{ac}+\frac{c^2}{ab}\ge \frac{(b+c)^2}{a(b+c)}=\frac{b+c}{a}$.

$\frac{b}{c+a}+\frac{c}{a+b}=\frac{b^2}{bc+ab}+\frac{c^2}{ac+bc}\ge \frac{(b+c)^2}{2bc+a(b+c)}\ge \frac{(b+c)^2}{\frac{(b+c)^2}{2}+a(b+c)}$

$=\frac{2(b+c)}{b+c+2a}=\frac{2\frac{b+c}{a}}{\frac{b+c}{a}+2}$.

Đặt $t=\frac{b+c}{a}$. Ta có: $t\ge 2$ và $P\ge f(t)=t+\sqrt{\frac{1}{t}}+\sqrt{\frac{2t}{t+2}}$.

Ta có:$f'(t)=1-\frac{1}{2t\sqrt{t}}+\frac{2}{(t+2)\sqrt{2t(t+2)}}=\frac{2t\sqrt{t}-1}{2t\sqrt{t}}+\frac{2}{(t+2)\sqrt{2t(t+2)}}>0\forall t\in (2;+\infty)$.

$\implies f(t)\ge f(2)=3+\frac{1}{\sqrt{2}}$.

Vậy $P\ge f(t)\ge 3+\frac{1}{\sqrt{2}}$. Dấu $=$ xảy ra tại $a=b=c\implies P_{min}=3+\frac{1}{\sqrt{2}}$ khi $a=b=c$.

Lời giải bài 24:

Ta có: $P=\frac{(a+b)^2}{c^2+4c(a+b)+4ab}\ge \frac{(a+b)^2}{c^2+4c(a+b)+(a+b)^2}\rightarrow M$.

Do $a,b,c\in [1,2]$ nên $a+b\ne 0$, nên chia tử và mẫu của $M$ cho $(a+b)^2$ ta được:

$M=\frac{1}{(\frac{c}{a+b})^2+4(\frac{c}{a+b})+1}=\frac{1}{t^2+4t+1}$ với $t=\frac{c}{a+b}$. Với $a,b,c\in [1,2]\iff t\in [\frac{1}{4},1]$.

Xét hàm số $f(t)=\frac{1}{t^2+4t+1}$ trên $[\frac{1}{4},1]$.

Ta có: $f'(t)=\frac{-2(t+2)}{(t^2+4t+1)^2}<0,\forall t\in [\frac{1}{4},1]\implies f'(t)$ nghịch biến trên $[\frac{1}{4},1]$,

Do đó: $t\le 1\implies f(t)\ge f(1)=\frac{1}{6}$.

Đẳng thức xảy ra khi $t=1\iff (a;b;c)=(1;1;2)$.

Vậy $P_{min}=\frac{1}{6}\iff (a;b;c)=(1;1;2)$ 



#53
tritanngo99

tritanngo99

    Đại úy

  • Điều hành viên THPT
  • 1644 Bài viết

Tiếp theo: 

Bài 25:Cho $x,y$ thỏa mãn: $x-3\sqrt{x+1}=3\sqrt{y+2}-y$. Tìm GTLN và GTNN của biểu thức: $P=x+y$.

Bài 26: Trong các nghiệm $(x;y)$ của hệ:$\left\{\begin{matrix} x+y\le 2\\ x^2+y^2+xy=3  \end{matrix}\right.$, hãy tìm nghiệm sao cho $x^2+y^2-xy$ đạt GTLN,GTNN.



#54
Dinh Xuan Hung

Dinh Xuan Hung

    Thành viên nổi bật 2015

  • Thành viên nổi bật 2016
  • 1396 Bài viết

Tiếp theo: 

Bài 25:Cho $x,y$ thỏa mãn: $x-3\sqrt{x+1}=3\sqrt{y+2}-y$. Tìm GTLN và GTNN của biểu thức: $P=x+y$.

Bài 26: Trong các nghiệm $(x;y)$ của hệ:$\left\{\begin{matrix} x+y\le 2\\ x^2+y^2+xy=3  \end{matrix}\right.$, hãy tìm nghiệm sao cho $x^2+y^2-xy$ đạt GTLN,GTNN.

Bài 25:

 

$\left ( \sqrt{x+1}+\sqrt{y+2} \right )^2\leq 2\left ( x+y+3 \right )\Leftrightarrow \frac{\left ( x+y \right )^2}{9}\leq 2(x+y)+6\Leftrightarrow (x+y)^2-18(x+y)-54\leq 0\Leftrightarrow 9-3\sqrt{15}\leq x+y\leq 9+3\sqrt{15}$



#55
tritanngo99

tritanngo99

    Đại úy

  • Điều hành viên THPT
  • 1644 Bài viết

Dưới đây là lời giải bài 25 và bài 26:

Lời giải bài 25:

Đặt $x+y=S$. Đặt $\sqrt{x+1}=a;\sqrt{y+2}=b$ thì $a,b\ge 0$ và $x=a^2-1;y=b^2-2$. Hệ trở thành:

$\left\{\begin{matrix} a^2+b^2-3(a+b)=3\\ a^2+b^2=S+3  \end{matrix}\right.$

$\iff$ $\left\{\begin{matrix} S+3-3(a+b)=3\\(a+b)^2-2ab=S+3  \end{matrix}\right.$

$\iff$ $\left\{\begin{matrix} a+b=\frac{S}{3}\\ ab=\frac{S^2-9S-27}{18}  \end{matrix}\right.$(I).

Hệ $(I)$ có nghiệm $(a;b)$ khi và chỉ khi :

$\left\{\begin{matrix} (\frac{S}{3})^2\ge 4*\frac{S^2-9S-27}{18}\\S\ge 0\\ S^2-9S-27\ge 0  \end{matrix}\right.$

$\iff \frac{9+3\sqrt{21}}{2}\le S\le 9+3\sqrt{15}$.

Vậy $max(x+y)=9+3\sqrt{15};min(x+y)=\frac{9+3\sqrt{21}}{2}$

Lời giải bài 26:

Đặt $a=x+y-2$ thì hệ tương đương với $\left\{\begin{matrix} x+y-2=a,a\le 0\\x^2+y^2+xy=3  \end{matrix}\right.$

Ta có:$\left\{\begin{matrix} x+y-2=a\\x^2+y^2+xy=3  \end{matrix}\right.$ $\iff$ $\left\{\begin{matrix} x+y=2+a\\(x+y)^2-xy=3  \end{matrix}\right.$

$\left\{\begin{matrix} x+y=2+a\\xy=(2+a)^2-3  \end{matrix}\right.$

Điều kiện đối với $a$ để hệ có nghiệm $(x;y)$ là:

$(2+a)^2\ge 4((a+2)^2-3)\iff (2+a)^2\le 4\iff -4\le a\le 0$(thỏa mãn điều kiện $a\le 0$).

Khi đó: $x^2+y^2-xy=x^2+y^2+xy-2xy=3-2((2+a)^2-3)=-2a^2-8a+1$.

Xét hàm số: $f(a)=-2a^2-8a+1,a\in [-4;0]$.

Lập BBT của hàm số này trên đoạn $[-4;0]$ ta được:

$min(x^2+y^2-xy)=1$ khi $a=0...or...a=-4$ tức là khi $x=y=1...or...x=y=-1$.

$max(x^2+y^2-xy)=9$ khi $a=-2$ tức là khi $x=\sqrt{3};y=-\sqrt{3}$ hoặc $x=-\sqrt{3};y=\sqrt{3}$ 



#56
tritanngo99

tritanngo99

    Đại úy

  • Điều hành viên THPT
  • 1644 Bài viết

Tiếp theo:

Bài 27: Cho $x,y,z$ là các số thực không âm. Chứng minh rằng:

$x^2(y+z-x)+y^2(x+z-y)+z^2(x+y-z)\le 3xyz$.

Bài 28: Chứng minh rằng nếu $a,b,c\in [1;2]$ thì $\frac{10a}{bc}+\frac{11b}{ca}+\frac{12c}{ab}\le \frac{69}{2}$ 



#57
nguyenhongsonk612

nguyenhongsonk612

    Thượng úy

  • Thành viên
  • 1451 Bài viết

Tiếp theo:

Bài 28: Chứng minh rằng nếu $a,b,c\in [1;2]$ thì $\frac{10a}{bc}+\frac{11b}{ca}+\frac{12c}{ab}\le \frac{69}{2}$ 

Thử làm xem có đúng không

Giải:

Đặt $f(a;b;c)=\frac{10a}{bc}+\frac{11b}{ca}+\frac{12c}{ab}$

$f(a;b;c)$ liên tục trên $[1;2]$

$f'(a)=\frac{10}{bc}-\frac{11b}{ca^2}-\frac{12c}{a^2b}$

$f''(a)=\frac{22b}{ca^3}+\frac{24c}{a^3b}>0$

$\Rightarrow $ Đồ thị hàm số $f(a)$ lõm trên $[1;2]$

 $\Rightarrow$ $\max f(a;b;c)=\max \left \{ f(1;b;c);f(2;b;c) \right \}$

Cứ tiếp tục làm tương tự như vậy ta được  

$\max f(a;b;c)=\max \left \{ f(1;1;1);f(1;1;2);f(1;2;1);f(2;1;1);f(1;2;2);f(2;2;1);f(2;1;2);f(2;2;2) \right \}=\frac{69}{2}$

Vậy ta có đpcm

Dấu "=" xảy ra khi $a=b=1;c=2$


Bài viết đã được chỉnh sửa nội dung bởi nguyenhongsonk612: 21-08-2016 - 21:41

"...Từ ngay ngày hôm nay tôi sẽ chăm chỉ học hành như Stardi, với đôi tay nắm chặt và hàm răng nghiến lại đầy quyết tâm. Tôi sẽ nỗ lực với toàn bộ trái tim và sức mạnh để hạ gục cơn buồn ngủ vào mỗi tối và thức dậy sớm vào mỗi sáng. Tôi sẽ vắt óc ra mà học và không nhân nhượng với sự lười biếng. Tôi có thể học đến phát bệnh miễn là thoát khỏi cuộc sống nhàm chán khiến mọi người và cả chính tôi mệt mỏi như thế này. Dũng cảm lên! Hãy bắt tay vào công việc với tất cả trái tim và khối óc. Làm việc để lấy lại niềm vui, lấy lại nụ cười trên môi thầy giáo và cái hôn chúc phúc của bố tôi. " (Trích "Những tấm lòng cao cả")

~O) 


#58
quochungtran

quochungtran

    Lính mới

  • Thành viên mới
  • 5 Bài viết

Bài 27  :                 Bài này có thể dùng kỹ thuật SOS.

    BĐT tương đương x3 + y+ z3 - 3xyz    $\geq$   x2(y+z) + y2(x+z) + z2(x+y) - 6xyz                     

                           

 hay ( x+ y +z)( (x-y)2 + (y-z)2 + (x-z)2 )   $\geq$   2 ( (x+y)(y+z)(x+z) - 8xyz )= 2 (  x(y-z)2 + y(x-z)2 + z(x-y)2 )

 

  $\sum$  (x-y)2 (x+y-z)   $\geq$   0.   Đặt Sx  = y + z -x . Sy = x+z - y . S= x + y - z. không mất tính tông quát giả sử          x $\geq$ y  $\geq$ z.

 

 Nên  Sy   > 0, ta có ( x- z)2   $\geq $ ( x - y)2 + ( y -  z)2 <=> (x-y)(y-z) $\geq$ 0.(đúng)

 

Vì vậy ta có Sx (y-z)2  +  Sy(x-z)2   + Sz (y-x)2  $\geq$ (y-x)2 ( Sz  +  Sy ) + (y-z)2 ( Sy + Sx ) $\geq$  0 (*)

   

 mà  Sz  +  Sy  = 2x >0  ,   Sy + Sx   =  2z > 0    suy ra (*)   đúng , suy ra dpcm.      

             

         Dấu bằng xảy ra khi và chỉ khi x=y=z                                       

                    


Bài viết đã được chỉnh sửa nội dung bởi quochungtran: 20-08-2016 - 23:19

~O)  ~O)  ~O)  :excl:  :excl:  :excl:  ~O)  ~O)  ~O)  :ukliam2:


#59
quochungtran

quochungtran

    Lính mới

  • Thành viên mới
  • 5 Bài viết

cho mình đề nghị 1 bài

 

Bài 29)     Giả sử x,y,z  $\geq$ 0 và x+y+z = 3 . hãy tìm GTNN của x4 + 2y4 + 3z4.

.


Bài viết đã được chỉnh sửa nội dung bởi quochungtran: 20-08-2016 - 23:01

~O)  ~O)  ~O)  :excl:  :excl:  :excl:  ~O)  ~O)  ~O)  :ukliam2:


#60
tritanngo99

tritanngo99

    Đại úy

  • Điều hành viên THPT
  • 1644 Bài viết

Dưới đây là lời giải bài 27 và bài 28:

Lời giải bài 27: Do $BDT$ thuần nhất nên nhờ chuẩn hóa ta có thể giả thiết thêm: $x+y+z=1$.

Thay $(y+z;z+x;x+y)\rightarrow (1-x;1-y;1-z)$ vào biểu thức ban đầu ta được:

$x^2(1-2x)+y^2(1-2y)+z^2(1-2z)\le 3xyz$.

$\iff x^2+y^2+z^2\le 2(x^3+y^3+z^3)+3xyz$.

$\iff (x+y)^2-2xy+z^2\le 2((x+y)^3-3xy(x+y)+z^3)+3xyz$.

$\iff (1-z)^2-2xy+z^2\le 2((1-z)^3-3xy(1-z)+z^3)+3xyz$

$\iff (9z-4)xy+4z^2-4z+1\ge 0$.

Đặt $t=xy,0\le xy\le (\frac{x+y}{2})^2=\frac{(1-z)^2}{4}$ và xét $f(t)=(9z-4)t+4z^2-4z+1$.

Ta có: $f(0)=4z^2-4z+1=(2z-1)^2\ge 0$

$f(\frac{(1-z)^2}{4})=\frac{z(3z-1)^2}{4}\ge 0$

$\implies f(t)\ge 0,\forall t\in [0;\frac{(1-z)^2}{4}]$.

Đẳng thức xảy ra $\iff x=y=z=\frac{1}{3}$ hoặc $x=y=\frac{1}{2};z=0$ và các hoán vị. Điều này tương đương với $a=b=c$ hoặc $a=b;c=0$ và các hoán vị của nó.

Bất đẳng thức được chứng minh hoàn tất.

Lời giải bài 28:

 Ta coi một trong ba số $a,b,c$ là một biến số của hàm số, chẳng hạn là $a$, khi đó ta đặt $x=a,x\in[1;2]$ và ta đi xét hàm số:

$f(x)=\frac{1}{x}(\frac{11b}{c}+\frac{12c}{b})+\frac{10x}{bc}$, đặt 
$\alpha=\frac{11b}{c}+\frac{12c}{b}=\frac{11b^2+12c^2}{bc};\beta=\frac{10}{bc}$.

Khi đó: $f'(x)=\frac{-\alpha}{x^2}+\beta=\frac{\beta x^2-\alpha}{x^2},f'(x)=0\iff x=\sqrt{\frac{\alpha}{\beta}}$.

Ta có: $\alpha=\frac{11b^2+12c^2}{bc}\ge \frac{33}{bc}>3*\frac{10}{bc}=3\beta\implies x=\sqrt{\frac{\alpha}{\beta}}>1$.

Như vậy ta luôn có: $f(x)\le max(f(1),f(2))=max(g(b),h(b))$, trong đó: $g(b)=f(1)=\frac{10}{bc}+\frac{11b}{c}+\frac{12c}{b}$ và:

$h(b)=f(2)=\frac{20}{bc}+\frac{11b}{2c}+\frac{6c}{b}$.

Ta xét tiếp $g(b)$ trên đoạn $[1;2]$ có: $g'(b)=\frac{-1}{b^2}(\frac{10}{c}+12c)+\frac{11}{c}=\frac{-1}{b^2}A+B$, trong đó:

$A=\frac{10}{c}+12c=\frac{10+12c^2}{c},B=\frac{11}{c}$ và có: $g'(b)=0\iff b=\sqrt{\frac{A}{B}}>1$.

Như vậy, $g(b)\le max(g(1),g(2))=max(\frac{21}{c},\frac{27}{c}+6c)$.

Xét lần nữa: $n(c)=\frac{21}{c}+12c$ và $m(c)=\frac{27}{c}+6c$ trên đoạn $[1;2]$ có $max_{[1;2]}(n(c),m(c))\le max(\frac{69}{2},33)=\frac{69}{2}$,

Từ đó suy ra: $g(b)\le \frac{69}{2}\forall b,c\in[1;2]$.

Xét tương tự đối với $h(b)\le max(h(1),h(2))=max(\frac{51}{2c}+6c,\frac{21}{c}+3c)\le max(\frac{63}{2},24)=\frac{69}{2}$.

Vậy $\frac{10a}{bc}+\frac{11b}{ca}+\frac{12c}{ab}\le \frac{69}{2}$, đẳng thức xảy ra khi và chỉ khi $a=b=1,c=2$. 

Ps: Đây là một trong những phương pháp hàm số khá hữu hiệu đối với những bài toán bị chặn khoảng.






0 người đang xem chủ đề

0 thành viên, 0 khách, 0 thành viên ẩn danh